Categories
Economists Harvard

Harvard. List of Ph.D. recipients in History and Political Science, 1873-1901

Before there was a department of political economy or economics at Harvard there was a Division of History and Political Science that continued on to become the Division of History, Government and Economics. Earlier student records for graduate students of economics as well as for the other departments were kept at this divisional and not departmental level. Altogether a total of 45 Ph.D. degrees were awarded at Harvard going up through 1901, not quite a third to men who were or became economists or economic historians.

The following list includes the activity of the Ph.D. alumni, presumably as of 1900-01.

____________________

DIVISION OF HISTORY AND POLITICAL SCIENCE
RECIPIENTS OF THE DEGREE OF PH.D., 1873-1901.

(* deceased)

Field. Date.
1. Charles Leavitt Beals Whitney* History. 1873.
2. Stuart Wood Pol. Sci. 1875.
3. James Laurence Laughlin
(Prof. Chicago)
History. 1876.
4. Henry Cabot Lodge
(Former Instr. Harv., now Senator)
History. 1876.
5. Ernest Young*
(Late Prof. Harvard)
History. 1876.
6. Freeman Snow*
(Late Instr. Harvard)
History. 1877.
7. Franklin Bartlett History. 1878.
8. Melville Madison Bigelow
(Prof. Boston Univ.)
History. 1878.
9. Edward Channing
(Prof. Harvard)
History. 1880.
10. Denman Waldo Ross
(Lecturer, Harvard)
History. 1880.
11. Samuel Eppes Turner*
(once Instr. Philips Exeter)
History. 1880.
12. Frank William Taussig
(Prof. Harvard)
Pol. Sci. 1883.
13. Andrew Fiske
(Lawyer)
History. 1886.
14. Charles William Colby
(Prof. McGill)
History. 1890.
15. Edson Leone Whitney
(Prof. Bezonia)
History. 1890.
16. Herman Vanderburg Ames
(Prof. Pennsylvania)
History. 1891.
17. Fred Emory Haynes
(Charity Work)
History. 1891.
18. Evarts Boutell Greene
(Prof. Illinois)
History. 1893.
19. Charles Luke Wells
(Recent Prof. Minnesota)
History. 1893.
20. Willian Edward Burghardt DuBois
(Prof. Atlanta)
Pol. Sci. 1895.
21. Kendric Charles Babcock
(Prof. California)
History. 1896.
22. Howard Hamblett Cook
(Statistician)
Pol. Sci. 1896.
23. Theodore Clarke Smith
(Prof. Ohio State Univer.)
Pol. Sci. 1896.
24. Guy Stevens Callender
(Prof. Bowdoin)
Pol. Sci. 1897.
25. Clyde Augustus Duniway
(Prof. Leland Stanford Univ.)
Pol. Sci. 1897.
26. Gaillard Thomas Lapsley
(Instr. Univ. Cali.)
History. 1897.
27. Charles Whitney Mixter
(Instr. Harvard)
Pol. Sci. 1897.
28. Oliver Mitchell Wentworth Sprague
(Instr. Harvard)
Pol. Sci. 1897.
29. George Ole Virtue
(Prof. Wisconsin Normal)
Pol. Sci. 1897.
30. Samuel Bannister Harding
(Prof. Indiana)
History. 1898.
31. James Sullivan, Jr.
(Instr. N.Y. [DeWitt Clinton] High School)
History. 1898.
32. Arthur Mayer Wolfson
(Instr. N.Y. [DeWitt Clinton] High School)
History. 1898.
33. Frederick Redman Clow
(Prof. Minnesota Normal)
Pol. Sci. 1899.
34. Arthur Lyons Cross
(Instr. Michigan)
History. 1899.
35. Louis Clinton Hatch. History. 1899.
36. Norman Maclaren Trenholme
(Instr. Pa. State College)
History. 1899.
37. Abram Piatt Andrew, Jr.
(Instr. Harvard)
Pol. Sci. 1900.
38. Sidney Bradshaw Fay
(Instr. Harvard)
History. 1900.
39. Carl Russell Fish
(Instr. Wisconsin)
History. 1900.
40. William Bennett Munro
(Instr. McGill)
Pol. Sci. 1900.
41. Subharama Swaminadhan Pol. Sci. 1900.
42. Don Carlos Barrett
(Prof. Haverford)
Pol. Sci. 1901.
43. Herbert Camp Marshall Pol. Sci. 1901.
44. Jonas Viles History. 1901.
45. Arthur Herbert Wilde
(Prof. Northeastern)
History. 1901.

Source: Harvard University Archives. Division of History, Government & Economics. PhD. Material. Box 1, Folder “PhD degrees conferred, 1873-1901 (Folder 1 of 2).”

Image Source: Harvard Square, ca. 1901-07. History Cambridge webpage “Postcard Series I: Harvard Square”.

Categories
Economics Programs Exam Questions Harvard

Harvard. Mid-year and Year-End Final Exams in Economics and Social Ethics, 1896-1897

 

The collection of transcribed Harvard semester examinations here at Economics in the Rear-view Mirror is growing slowly. This post adds the exam questions from 1896-1897 for all the economics courses and for Francis Peabody’s philosophy course “The Ethics of the Social Questions”. 

_____________________

From the Preface to the Announcements
for 1896-97

The courses primarily for Undergraduates are open (and in some cases recommended) to Graduate Students, but are not ordinarily counted towards any of the higher degrees. The courses for Graduates and Undergraduates are, under certain limitations, open to any properly qualified student. To the courses primarily for Graduates Undergraduates are admitted only on the recommendation of the Instructor.

No starred (*) course and no course of research can be taken without the previous consent of the Instructor….

By recent action of the Governing Boards, the requirement of two years of residence at this University of a candidate for the degree of Doctor of Philosophy or Doctor of Science has been rescinded. The minimum requirement of residence in now one year, this period being fixed for all degrees by the Statutes. It is not the purpose of the change thus made to lower the standards for these two degrees; but only to reduce the amount of compulsory residence at this University. (April, 1896).

Source: Harvard University. Announcement of the Courses of Instruction Provided by the Faculty of Arts and Sciences for the Academic Year 1896-1897, pp. iii-iv.

_____________________

Philosophy 5.
The Ethics of the Social Questions.
1896-1897

Course Announcement

[Philosophy] 5. The Ethics of the Social Questions. — The problems of Poor-Relief, the Family, Temperance, and various phases of the Labor Question in the light of ethical theory. — Lectures, special researches, and required reading. Tu., Th., Sat., at 10. Professor Peabody.

Source: Harvard University. Announcement of the Courses of Instruction Provided by the Faculty of Arts and Sciences for the Academic Year 1896-1897, p. 26.

Course Enrollment

[Philosophy] 5. Professor Peabody. — The Ethics of the Social Questions. — The problems of Poor-Relief, the Family, Temperance, and various phases of the Labor Question in the light of ethical theory. — Lectures, special researches, and required reading. 3 hours.

Total 56: 1 Graduate, 31 Seniors, 10 Juniors, 3 Sophomores, 11 Others.

Source: Harvard University. Report of the President of Harvard College 1896-1897, p. 66.

*  *  *  *  *
[Mid-year examination,
still to be added]

*  *  *  *  *

Philosophy 5.
THE ETHICS OF THE SOCIAL QUESTIONS.
Year-End Examination,
1896-97

[This paper should be considered as a whole. The time should not be exhausted in answering a few questions, but such limit should be given to each answer as will permit the answering of all the questions in the time assigned.]

  1. Indicate, briefly, the place in the history of the modern Labor Question of:
    Chalmers;
    Von Ketteler;
    Lassalle;
    The Rochdale Pioneers;
    Carlyle;
  2. Ruskin as an Economist.
  3. The anarchist’s criticism of the socialist; the socialist’s criticism of the anarchist, and the communist as he is criticised by both.
  4. What do you understand to be the “quintessence” of socialism, as expounded by Schäffle; and what criticisms on this whole social programme appear to you most serious?
  5. Arbitration and conciliation — their differences, varieties, advantages, and limitations.
  6. The history of co-operation in Great Britain, its fortunes in the United States, and the conditions of its success.
  7. Various types of industrial partnership, — their special advantages and limitations.
  8. The Scandinavian Licensing System compared with the present Massachusetts Liquor Law. (Fanshawe, 187-229.)
  9. Sum up, briefly, the general doctrine of social duty which our study of various social questions is intended to illustrate.

Source: Harvard University Archives. Harvard University Examination Papers, 1873-1915. Box 4. Bound volume: Examination Papers 1896-97. Papers Set for Final Examinations in Philosophy, History, Government, Economics, Fine Arts, Architecture, and Music in Harvard College, June 1897, p. 7.

_____________________

ECONOMICS.
Primarily for Undergraduates.

_____________________

Economics 1.
Outlines of Economics,
1896-97

Course Announcement

[Economics] 1. Outlines of Economics. — Principles of Political Economy. — Lectures on Economic Development, Social Questions, and Financial Legislation. Mon., Wed., Fri., at 9. Professors Taussig and Ashley, Asst. Professor Edward Cummings, and Dr. John Cummings.

Source: Harvard University. Announcement of the Courses of Instruction Provided by the Faculty of Arts and Sciences for the Academic Year 1896-1897, p. 33.

Course Enrollment

[Economics] 1. Professors Taussig and Ashley, Asst. Professor Edward Cummings, and Dr. John Cummings. Outlines of Economics. — Principles of Political Economy. — Lectures on Economic Development, Social Questions, and Financial Legislation. 3 hours.

Total 464: 1 Graduate, 40 Seniors, 131 Juniors, 235 Sophomores, 12 Freshmen, 45 Others.

Source: Harvard University. Report of the President of Harvard College 1896-1897, p. 65.

Economics 1.
Mid-year Examination,
1896-97

  1. “Productive labor is that which produces utilities fixed and
  2. embodied in material objects. All other labor, however useful, is classed as unproductive.” Why? or why not?
  3. Capital is the result of saving: capital is produced by labor; all capital is consumed. Can you reconcile these propositions?
  4. “Those circumstances of a country, in which population can with impunity increase at its utmost rate, are rare and transitory.” What are they? and why rare? What is the utmost rate?
  5. Give examples of (a) differences in wages arising from different degrees of attractiveness in different employments; (b) differences arising from natural monopolies.
  6. Suppose a tax were imposed on land precisely equal to the economic rent paid for its use; could the owner of the land shift the tax to his tenant by charging a higher rent than before?
  7. What is meant when it is said that rent does not enter into the cost of production?
  8. It is said to be immaterial whether a community has a large or a small stock of money; and it is said to be harmful for a community to resort to inconvertible paper. Can both of these propositions be sound?
  9. “With enormous shortsightedness, the people of the United States send abroad every year over one hundred millions of dollars, with which to pay for sugar which might have been produced at home.” Why, or why not, is there shortsightedness in this operation?
  10. Wherein does a country gain, if other countries demand more of its exports?
  11. In the stationary state. as described by Mill, what determines the rate of interest? the rate of wages?

Source: Harvard University Archives. [Examinations] Scrapbook of F. W. Taussig, p. 58.

Economics 1.
Year-end Examination,
1896-97

[Answer nine questions, selecting at least one from each of the four groups. Arrange your answers strictly in the order of the questions selected. Give your reasons in all cases.]

I.

  1. Mention a case in which the income received for the use of a piece of real estate is to be regarded as rent; one in which it is to be regarded as interest; and one in which the classification would be doubtful.
  2. Mention a commodity whose value is permanently governed by cost of production; one whose value is permanently governed by the equation of demand and supply; and one whose value is permanently affected by both causes.
  3. Does the gain from foreign trade arise from the sale of exports? from the purchase of imports?

II.

  1. Is the law of Rent stated by Mill applicable either to the German peasant of the early part of the century or to the American farmer of to-day? Give your reasons.
  2. What are the functions of the entrepreneur? Give some account of the origin of the entrepreneur
  3. Illustrate the influence of the feeling of nationality as an economic factor.

III.

  1. How far have English trade unions tried to control (a) the wages of labor? (b) the supply of labor? In what respects have they been most useful?
  2. What do you understand by the rise of the modern factory system? In what sense is this system responsible for what is known as the labor problem?
  3. If all the productive and distributive business of a community were in the hands of cooperative societies would the labor question cease to exist?

IV.

  1. Point out wherein the deposits of a bank resemble its notes, and wherein they differ from its notes; and consider why one or the other should be regarded as part of the circulating medium of the community.
  2. What was the amount outstanding (in round numbers) of legal-tender notes in 1867? in 1877? in 1897? What were the laws under which the notes outstanding at those several dates had been issued, and what was the state of the legislation then in force for their redemption?
  3. Wherein does the Reichsbank of Germany, as to its management of notes and deposits, resemble the Bank of France? Wherein the Bank of England?

Source: Harvard University Archives. Harvard University Examination Papers, 1873-1915. Box 4. Bound volume: Examination Papers 1896-97. Papers Set for Final Examinations in Philosophy, History, Government, Economics, Fine Arts, Architecture, and Music in Harvard College, June 1897, pp. 37-38.

_____________________

ECONOMICS
For Graduates and Undergraduates.

The Courses for Graduates and Undergraduates are open to students who have passed satisfactorily in Course 1. Courses 5, 6, 7, 8, and 9 are also open to Juniors and Seniors of good rank who take Course 1 at the same time; and Course 11 is open to students who have taken either Economics 1 or History 1.

_____________________

Economics 10.
The Mediaeval Economic History of Europe

[[Economics] *10. The Mediaeval Economic History of Europe. Tu., Th., (and at the pleasure of the instructor) Sat., at 12. Professor Ashley.]

Omitted in 1896-97. Courses 10 and 11 are usually given in alternate years.

Source: Harvard University. Announcement of the Courses of Instruction Provided by the Faculty of Arts and Sciences for the Academic Year 1896-1897, p. 33.

_____________________

Economics 11.
The Modern Economic History of Europe and America (from 1600),
1896-97

Course Announcement

[Economics] 11. The Modern Economic History of Europe and America (from 1600). Tu., Th., (and at the pleasure of the instructor) Sat., at 12. Professor Ashley.

Courses 10 and 11 are usually given in alternate years.

Source: Harvard University. Announcement of the Courses of Instruction Provided by the Faculty of Arts and Sciences for the Academic Year 1896-1897, p. 33.

Course Enrollment

[Economics] 11. Professor Ashley. The Modern Economic History of Europe and America (from 1600). 2 hours.

Total 29: 16 Graduates, 6 Seniors, 1 Junior, 1 Sophomore, 5 Others.

Source: Harvard University. Report of the President of Harvard College 1896-1897, p. 65.

Economics 11.
Mid-Year Examination,
1896-97

N. B. — Not more than eight questions must be attempted.

  1. Give some account of Asiatic commerce in the middle ages.
  2. Enumerate very briefly some of the reasons for which Genoa deserves attention in economic history.
  3. “The mooste part of the lordes have enclosed their demeyn lands and meadows and kept them in severalties.” (Fitzherbert, c. 1530). Explain the nature and effects of the action here described.
  4. What features, if any, were common to the Peasant Risings in the different countries of Western Europe in the fourteenth, fifteenth, and sixteenth centuries?
  5. Compare the action of the English government in relation to industry in the sixteenth century with that of the French government.
  6. “Our merchants may do well to provide for the Russians such wares as the Dutch nation doth serve them of.” Explain and comment.
  7. Describe the industrial condition of Norfolk in the time of Defoe.
  8. Explain, with illustrations, what is meant by the “territorial” period in German economic development.
  9. What impressions do you derive from Defoe’s Essay upon Projects as to the constitution, temper and interests of the business circles of London in his time?
  10. Explain the following terms Droit de vaine pâture, Société en commandite, Niederlegung von Hufen, Hausindustrie, Fondaco.
  11. Give a critical account of any really important work (not on the printed list) of which you have read any considerable portion in connection with this course.

Source: Harvard University Archives. Harvard University Mid-Year Examination, 1852-1943. Box 4. Bound volume: Examination Papers, Mid-Years 1896-97.

Economics 11.
Year-end Examination,
1896-97

N.B. – Not more than eight questions must be attempted.

  1. “The fifteenth century was the golden age of the English labourer.” What is the evidence for that contention? How is that evidence to be interpreted?
  2. Explain the part played by Hamburg in the economic history of the sixteenth and seventeenth centuries.
  3. Give some account of the history, since the massacre of Amboyna, of the Dutch East Indian Empire.
  4. Indicate very briefly the chief points in Schmoller’s account of mercantilist policy. In what directions does it seem open to criticism?
  5. State the causes and criticize the alleged consequences of the drain of specie from the English colonies in the eighteenth century.
    [Not to be taken by those who have written theses on the Navigation Act.]
  6. “English industries could not have advanced so rapidly without protection.” Examine this statement.
  7. Describe the main features of English industrial life in the early part of the eighteenth century.
  8. What changes did the French Revolution make in the position of the rural population of France.
  9. Discuss the application of the Infant Industries argument to the United States during the early decades of the present century.
  10. Give some account of the competition between railroads and canals as means of transportation.
  11. What exactly were the English “Corn Laws,” repealed in 1846? Have the anticipations of Cobden been realized?
  12. “You made me look rather a fool, Arminius,” I began, “by what you primed me with in Germany last year about Stein settling your land question.” “I dare say you looked a fool,” says my Prussian boor, “but what did I tell you?” “Why,” says I, “you told me Stein had settled a land question like the Irish land question, and I said so in the Cornhill Magazine, and now the matter has come up again by Mr. Bright talking at Dublin of what Stein did, and it turns out he settled nothing like the Irish land question at all, but only a sort of title-commutation affair.” “Who says that?” asked Arminius. “A very able writer in the Times,” I replied. — May we have your opinion?

Source: Harvard University Archives. Harvard University Examination Papers, 1873-1915. Box 4. Bound volume: Examination Papers 1896-97. Papers Set for Final Examinations in Philosophy, History, Government, Economics, Fine Arts, Architecture, and Music in Harvard College, June 1897, p. 46.

_____________________

Economics 15.
The History and Literature of Economics to the Middle of the Nineteenth Century,
1896-97

Course Announcement

[Economics] *15. The History and Literature of Economics to the Middle of the Nineteenth Century. Mon., Wed., (and at the pleasure of the instructor) Fri., at 12. Professor Ashley.

Source: Harvard University. Announcement of the Courses of Instruction Provided by the Faculty of Arts and Sciences for the Academic Year 1896-1897, p. 34.

Course Enrollment

[Economics] 15. Professor Ashley. — The History and Literature of Economics to the Middle of the Nineteenth Century. 2 hours.

Total 14: 10 Graduates, 2 Seniors, 2 Juniors.

Source: Harvard University. Report of the President of Harvard College 1896-1897, p. 65.

Economics 15.
Mid-Year Examination,
1896-97

N.B. — Not more than eight questions must be attempted.

  1. Explain the influence of the Greek conception of the State upon the economic speculation of Greek philosophers.
  2. “The sacredness of property is a notion far more fixed in modern than in ancient times.” Comment upon this remark
  3. Explain and illustrate the influence of the example of Sparta on Greek social thought.
  4. In what sense is it true that Plato anticipated Adam Smith’s teaching concerning division of labour?
  5. In what sense is it true that Aristotle anticipated the modern distinction between Value in Use and Value in Exchange.
  6. Set forth briefly Aristotle’s doctrine as to Chrematistic.
  7. Discuss the question as to whether Christianity destroyed slavery.
  8. What sanction, if any, has Socialism or Communism in the teaching of the Christian Fathers?
  9. Trace the early history of the doctrine of “interest” in the original sense of that word.
  10. Distinguish between the various senses attached to the term Mercantilism. Which do you think most convenient?
  11. What ideas prominent in modern Protectionist argument are absent from Mercantilism as represented by Mun?

Source: Harvard University Archives. Harvard University Mid-Year Examination, 1852-1943. Box 4. Bound volume: Examination Papers, Mid-Years 1896-97.

Economics 15.
Year-End Examination,
1896-97

N.B. — Not more than eight questions must be attempted.

  1. Mention some of the practical questions which called forth economic pamphlets in the period 1650-1700, and give some illustrations.
  2. Illustrate and criticize the attitude of the mercantilist writers towards the regulation of internal industry.
  3. Explain the relation of Locke’s doctrine of price to the immediate purpose of his Considerations.
  4. Describe, as definitely as possible, the relation of the Physiocrats to the other reforming or revolutionary movements of their time.
  5. Criticize Adam Smith’s criticism of Physiocratic doctrine.
  6. What elements in his teaching do you conceive Adam Smith to have derived from Hume?
  7. What does Smith mean by “the component parts of the price of commodities”?
  8. Compare Smith’s definition of capital with that of John Stuart Mill.
  9. What does Smith mean by “the natural rates of wages”?
  10. What bearing has the teaching of Malthus on “Socialism.” Explain in your answer what sort of “Socialism” you have in mind, and mention the sources whence you have derived your impression of it.

Source: Harvard University Archives. Harvard University Examination Papers, 1873-1915. Box 4. Bound volume: Examination Papers 1896-97. Papers Set for Final Examinations in Philosophy, History, Government, Economics, Fine Arts, Architecture, and Music in Harvard College, June 1897, pp. 50-51.

_____________________

Economics 2.
Economic Theory from the Middle of the Nineteenth Century to the Present Time,
1896-97

Course Announcement

[Economics] *2. Economic Theory from the Middle of the Nineteenth Century to the Present Time. — English Writers. — The Austrian School. Mon., Wed., Fri., at 2.30. Professor Taussig.

Source: Harvard University. Announcement of the Courses of Instruction Provided by the Faculty of Arts and Sciences for the Academic Year 1896-1897, p. 34.

Course Enrollment

[Economics] 2. Professor Taussig. — Economic Theory from the Middle of the Nineteenth Century to the Present Time. — English Writers. — The Austrian School. 3 hours.

Total 42: 12 Graduates, 12 Seniors, 13 Juniors, 2 Sophomores, 3 Other.

Source:   Harvard University, Annual Reports of the President and Treasurer of Harvard College, 1896-97, p. 65.

Economics 2.
Mid-Year Examination,
1896-97

  1. “According to Ricardo, the exchange value of commodities contains neither return to capital nor rent, but simply labor.” Why? or why not?
  2. Sketch concisely the development of the general theory of value at the hands of Ricardo, Mill, Cairnes.
  3. “Skill, as skill, produces no effect on value; in other words, commodities do not under any circumstances exchange for each other in proportion to the degree of skill bestowed on them. Skill, though in itself inoperative on value, nevertheless affects it indirectly in two distinct ways; first, where competition is effective among producers, through the cost which must be undergone in acquiring the skill; . . . and secondly, in the absence of competition, through the principle of monopoly.” — Cairnes.
    Explain and illustrate.
  4. “If there really was a national fund the whole of which must necessarily be applied to the payment of wages, that fund could be no other than an aggregate of smaller similar funds possessed by the several individuals who compose the employing part of the nation. Does, then, any individual employer, possess any such fund? Is there any specific portion of any individual’s capital which the owner must necessarily expend upon labour? . . . May he not spend more or less on his family and himself, according to his fancy, — in the one case having more, in the other less, left for the conduct of his business? And of what is left, does he or can he determine beforehand how much shall be laid out on buildings, how much on materials, how much on labour? . . . Be it observed, fixity of definiteness is the very essence of the supposed wages-fund. No one denies that some amount or other must within any given period be disbursed in the form of wages. The only question is, whether that amount be determinate or indeterminate.” — Thornton, On Labour.
    State carefully, and consider critically, the answers Cairnes made to these questions.
  5. Would you accede to the statement that “President Walker’s theory is, in reality, not a theory of manager’s earnings at all, but a theory of differences in manager’s earnings”?
  6. “For an understanding of the machinery by which distribution is accomplished, the classification of sources of income should thus be different from that to be adopted for an explanation of the fundamental causes.” — Taussig.
    Wherein different?
  7. Explain what is meant by Consumer’s Rent; and consider how its significance is affected by inequalities in wealth.
  8. “As a rule, the poorer soils rise in value relatively to the richer, as the pressure of population increases.” — Marshall. Why?

Source: Harvard University Archives. Harvard University Mid-Year Examination, 1852-1943. Box 4. Bound volume: Examination Papers, Mid-Years 1896-97.

Economics 2.
Year-end Examination,
1896-97

  1. Do you believe that a permanent gain for the theory of wages has been made by Walker’s discussion of that subject? If so, wherein? if not, why not?
  2. Does Marshall’s analysis of the different grades of labor, and of the barriers between them, differ in essentials from Cairnes’s? from Mill’s?
  3. Explain what “quasi-rent” is, wherein it differs from true rent, wherein resembles true rent; and state whether the conception seems to you a helpful one, deserving to be permanently embodied in economic theory.
  4. What do you conceive the difference to be between what Walker calls “current product,” Marshall “the national dividend,” and the instructor in the course “real income”?
  5. On what grounds does Marshall maintain that “the extra income earned by natural abilities may be regarded as a rent, when we are considering the sources of the income of individuals, but not with reference to the normal earnings of a trade”? What is your own opinion?
  6. “The attribute of normal value implies systematic and continuous production.” Cairnes. Would Böhm-Bawerk accede to this proposition? Why, or why not? Give your own opinion.
  7. Explain what Böhm-Bawerk means by (subjective) “value”; and consider his analysis of the relation between value and cost.
  8. Enumerate the grounds on which Böhm-Bawerk maintains that “present goods have greater value than future goods of like kind and quantity”; consider to which of these grounds he gives most attention; and give your opinion as to the justice of this emphasis.

Source: Harvard University Archives. Harvard University Examination Papers, 1873-1915. Box 4. Bound volume: Examination Papers 1896-97. Papers Set for Final Examinations in Philosophy, History, Government, Economics, Fine Arts, Architecture, and Music in Harvard College, June 1897, p. 38. Previously transcribed: https://www.irwincollier.com/harvard-history-of-economic-theory-final-exam-questions-taussig-1897-1900/

_____________________

Economics 13.
Scope and Method in Economic Theory and Investigation

[*13 hf. Scope and Method in Economic Theory and Investigation. Half-course. Professor Taussig.]

Omitted in 1896-97; to be given in 1897-98.

Source: Harvard University. Announcement of the Courses of Instruction Provided by the Faculty of Arts and Sciences for the Academic Year 1896-1897, p. 34.

_____________________

Economics 3.
The Principles of Sociology.
1896-97

Course Announcement

[Economics] *3. The Principles of Sociology. —Development of the Modern State, and of its Social Functions. Mon., Wed., (and at the pleasure of the instructor) Fri., at 1.30. Asst. Professor Edward Cummings.

Source: Harvard University. Announcement of the Courses of Instruction Provided by the Faculty of Arts and Sciences for the Academic Year 1896-1897, p. 34.

Course Enrollment

[Economics] 3. Asst. Professor Edward Cummings. — The Principles of Sociology. — Development of the Modern State, and of its Social Functions. 2 hours.

Total 47: 6 Graduates, 23 Seniors, 13 Juniors, 2 Sophomores, 3 Other.

Source:   Harvard University, Annual Reports of the President and Treasurer of Harvard College, 1896-97, p. 65.

Economics 3.
Mid-Year Examination,
1896-97

(It is the purpose of the following questions to elicit first, an intelligent statement of the gist of what has been read or discussed; second, a free statement of an intelligent opinions you may be forming for yourselves.
Arrange your answers in the order in which the questions stand. Limit the discussion of each question to about an hour.)

I.

A critical comparison of Mr. Giddings’ treatise on the principles of Sociology with that of Mr. Spencer:—
(a) In regard to method, arrangement, and terminology :
(b) In regard to fundamental resemblances in the theories presented:
(c) In regard to supposed differences

II.

“Next in order come the problems of the social consciousness, or social mind, including its content of common memories and ideas, its aspirations and its volition.” What explanation of the phenomena in question is offered by (a) Giddings; (b) Spencer; (c) Durkheim; (d) Tarde? What is your opinion of the relative merits and the practical bearing of the several explanations?

IlI.

The significance and the function of the family (a) in the earlier and (b) in the later phases of social evolution.
(c) If you still have time, give some account of the successive theories and of the present state and significance of the controversy in regard to early forms of marriage.

Source: Harvard University Archives. Harvard University Mid-Year Examination, 1852-1943. Box 4. Bound volume: Examination Papers, Mid-Years 1896-97.

Economics 3.
Year-end Examination,
1896-97]

[Answer the questions in the order in which they stand. Give an hour to each group.]

I.

Discuss the conceptions of progress found in the following authors: Spencer, Comte, Giddings, Kidd, Kelly. Bluntschli. Indicate in each case (a) the nature of progress, (b) the criteria, (c) the chief causes.

What do you mean by progress?

What evidence of progress do you find in the historical sequence of the various forms of political organization which have lead to the modern state? Illustrate carefully.

Name and classify the principal types of political organization. Indicate briefly the social and industrial characteristics of each type.

II.

Give a critical summary of the views of Haycraft in Darwinism and Race Progress.

By what other writers, ancient or modern, have similar views been urged?

What importance do you attach to this school of thought?

What is the practical bearing of such views upon (a) the problems of scientific philanthropy and the treatment of defective and criminal classes? (b Upon socialism?

III.

State and criticise Bluntschli’s theory as to the nature and functions of the State, — the relation of the State to society and to the individual.

Compare Bluntschli’s theory with that of other writers, — Pollock, Spencer, Ritchie, Giddings.

What in your opinion are the merits and the defects of Bluntschli’s treatise?

Source: Harvard University Archives. Harvard University Examination Papers, 1873-1915. Box 4. Bound volume: Examination Papers 1896-97. Papers Set for Final Examinations in Philosophy, History, Government, Economics, Fine Arts, Architecture, and Music in Harvard College, June 1897, p. 39.

_____________________

Economics 14.
Socialism and Communism.

Course Announcement

[Economics] *14. Socialism and Communism. — History and Literature. Tu., Th., (and at the pleasure of the instructor) Sat., at 9. Asst. Professor Edward Cummings.

Source: Harvard University. Announcement of the Courses of Instruction Provided by the Faculty of Arts and Sciences for the Academic Year 1896-1897, p. 34.

Course Enrollment

[Economics] 14. Asst. Professor Edward Cummings.—Communism and Socialism.—History and Literature. 2 hours.

Total 13: 10 Seniors, 2 Juniors, 1 Sophomore.

Source: Harvard University. Report of the President of Harvard College, 1896-97, p. 65.

Economics 14.
Mid-Year Examination,
1896-97

(Arrange your answers in the order of the questions. Omit one.)

  1. The different senses in which the word Socialism is used. Where do you intend to draw the line between Socialism proper, and familiar forms of government interference and control – such as factory legislation, municipal water works, and government postal, telegraph or railroad services? Why?
  2. “National communism has been confused with the common ownership of the family; tenure in common has been confused with ownership in common; agrarian communism with village commons.” Discuss the evidence.
  3. “Just as Plato had his Republic, Campanella his City of the Sun, and Sir Thomas More his Utopia, St. Simon his Industrial System, and Fourier his ideal Phalanstery…. But the common criticism of Socialism has not yet noted the change, and continues to deal with the obsolete Utopias of the pre—evolutionary age.” What do you conceive to be the character of the change referred to? How far did earlier Utopias anticipate the ideals of the modern social democracy?
  4. What indication of Socialistic tendencies are to be found in the discipline of the Christian church? Explain the triple contract and its bearing on the doctrine of the usury.
  5. The contributions of Greek writers to the development of economic thought.
  6. To what extent are the theories of Karl Marx indebted to earlier writers in the 19th-century?
  7. How far are the economic series of (a) Lasalle, (b) Marx related to the theories of the so-called orthodox Economists? Explain critically.
  8. How far do you trace the influence of historical conditions in the social philosophies of Plato, More, Bacon, Rousseau, St. Simon, Karl Marx?

Source: Harvard University Archives. Harvard University Mid-Year Examination, 1852-1943. Box 4. Bound volume: Examination Papers, Mid-Years 1896-97.
Previously transcribed: https://www.irwincollier.com/harvard-exams-and-enrollment-for-economics-of-socialism-and-communism-edward-cummings-1893-1900/

Economics 14.
Year-end Examination,
1896-97

I.

  1. “The figures of Cardinal Manning and Monsignor Bagshawe in England, of De Mun in France, of Decurtins in Switzerland, of Abbé Hitz in Germany, and of Von Vogelsang in Austria, will ever stand apart as State Socialists who, while looking to the Church for moral reform, expected and wished all economic reforms to come from the State alone.”
    Give some account of the leaders mentioned, and discuss the peculiarities of so-called Catholic Socialism in each country.
  2. How far does the programme of “Catholic Socialism” in different countries harmonize with the programme of the German Social Democratic party.
  3. State carefully what has been the attitude of the Vatican towards Socialism? What are the personal views of Leo XIll?

II.

  1. Describe the origin, development, fortunes and present strength of the Social Democratic party in Germany, — with special reference to the Eisenach, Gotha and Erfurt programmes.
  2. State and criticise Marx’s Theory of Surplus Value. Explain carefully the formula

\left( S=P_{n}\times \frac{s\ l\  t}{n\  l\  t} \right)

  1. What are Schäffle’s chief criticisms of the Socialistic State?

III.

[Take one question.]

  1. “Though social conflicts are as old as civilization itself, Socialism as we now understand it is of scientific origin, and essentially modern.” State carefully your reasons for agreeing or disagreeing.
  2. What are the characteristics of modern Utopian ideals, as contrasted with the ideals embodied in earlier literature of the kind? Contrast Bellamy, Hertzka, Morris.
  3. Trace in a general way the influence of socialistic doctrines in the establishment of socialistic and communistic societies in the United States. What light has experiment thrown upon socialistic and communistic ideals?
  4. State accurately the reading you have done in this course during the second half-year. Give a careful summary of the views of the author you recently selected for your special reading.

Source: Harvard University Archives. Harvard University Examination Papers, 1873-1915. Box 4. Bound volume: Examination Papers 1896-97. Papers Set for Final Examinations in Philosophy, History, Government, Economics, Fine Arts, Architecture, and Music in Harvard College, June 1897, p. 49-50.

_____________________

Economics 9.
The Labor Question in Europe and the United States

Course Announcement

[Economics] 9. The Labor Question in Europe and the United States. — The Social and Economic Condition of Workingmen. Tu., Th., Sat., at 10. Asst. Professor Edward Cummings and Dr. John Cummings.

Source: Harvard University. Announcement of the Courses of Instruction Provided by the Faculty of Arts and Sciences for the Academic Year 1896-1897, p. 34.

Course Enrollment

[Economics] 9. Asst. Professor Edward Cummings and Dr. John Cummings. — The Labor Question in Europe and the United States. — The Social and Economic Condition of Workingmen. 3 hours.

Total 50: 5 Graduates, 9 Seniors, 27 Juniors, 7 Sophomores, 2 Others.

Source:   Harvard University, Annual Reports of the President and Treasurer of Harvard College, 1896-97, p. 65.

Economics 9.
Mid-Year Examination,
1896-97

Divide your time equally between A and B.

A.

  1. The labor problem and the unity of the labor movement as manifested in trade unionism, co-operation, and socialism.
  2. How, if at all, has the introduction of machinery directly modified methods of industrial remuneration?
    What are the several bases for determining remuneration?
    Explain the unpopularity, (a) with employers and (b) with employees, of certain methods of industrial remuneration, and, if you can, offer some general principle justifying the adoption of one method rather than another.
    Tell in what way, if at all, the amount of remuneration is affected by the method of remuneration.

B.

Take six.

  1. The “Old” and the “New” Trade-Unionism. Give an account of growth of the English Trade Unions.
  2. What social and economic motives have contributed to the growth of modern cities? Explain so far as you are able the migratory movements which have led to the aggregation of population within certain industrial centres.
  3. Define: “nibbling”; “lump system “; “pay-as-you-please” piece work; the “lump of labor” theory; “chasing”; “collective gain-sharing”; the “plus” system; “butty-gangs”; “tut-work”; “working in pocket”; “garret master”: “product-sharing”; “bribe participation.”
  4. How far are the various forms of profit-sharing “sops to Cerberus”? What is the essential difference between a profit-sharing firm and a co-operative association? How far does industrial co-operation enable workmen to become their own employers?
  5. To what extent is the development of modern machine industry dependent upon the location of the world’s coal fields?
  6. What are the economic and social conditions of industrial depressions?
  7. Are the evils of sweating due to underpay, to overwork, or to the method of remuneration? Are they peculiar to some particular method of remuneration?
  8. The methods of estimating the annual revenue of Great Britain and its distribution.
  9. Has the introduction of machinery lessened the demand for labor?

Source: Harvard University Archives. Harvard University Mid-Year Examination, 1852-1943. Box 4. Bound volume: Examination Papers, Mid-Years 1896-97.

Economics 9.
Year-end Examination,
1896-97

I.

  1. State the subject of your special research during the second half-year. How far were you successful in getting material from (a) newspapers, (b) magazines, (c) books, (d) other sources? What general results did you reach?
  2. Explain the essential difference between a socialistic policy and what Schäffle calls a “positive social policy.” What, according to Schäffle, are legitimate objects of protective legislation? Give some recount of German protective legislation, and the “factory labor” to which this legislation applies.
  3. What according to Mallock, determines the minimum wage in any occupation? How far is this minimum rate subject to legislative or other control? Has the introduction of machinery tended to raise or lower this rate? To increase or to lessen the proportion of wages to product?
  4. What do you understand by the statement that “today the labor contract is perfectly free”? Discuss the legality of (a) strikes; (b) boycots; (c) intimidation; (d) “molesting.”
  5. Describe accurately the German compulsory system of old-age insurance. State the precise arguments which are urged for and against the adoption of some such system in England. How far may the German system be said to be the product of peculiar local conditions?

II.

[Omit two questions.]

  1. Describe some of the chief agencies and methods devised for dealing with the unemployed. How far have they been successful?
  2. Describe the strength, composition and programme of the labor party in Belgium, and its relation to trade-unions, cooperation and socialism. Contrast the situation in Belgium with that in Germany.
  3. Show in what respects the general policy of France towards Associations of workingmen during the present century has differed from that of England. Indicate briefly the effects of this policy upon trade-unions, coöperation, etc.
  4. Discuss, the growth, character, programme and strength of existing labor organizations in the United States, — contrasting the situation in this country with the situation in England and explaining differences.
  5. By what peculiar local conditions in each case do you account for the success or the failure of the coöperative movements in (a) England, (b) France, (c) the United States? What do you predict for the future of coöperation?

Source: Harvard University Archives. Harvard University Examination Papers, 1873-1915. Box 4. Bound volume: Examination Papers 1896-97. Papers Set for Final Examinations in Philosophy, History, Government, Economics, Fine Arts, Architecture, and Music in Harvard College, June 1897, pp. 44-45.

_____________________

Economics 4.
Theory and Methods of Statistics,
1896-97

Course Announcement

[Economics] *4, Theory and Methods of Statistics. — Applications to Economic and Social Questions. —  Studies in the Movement of Population. Mon., Wed., Fri., at 11. Dr. John Cummings.

Source: Harvard University. Announcement of the Courses of Instruction Provided by the Faculty of Arts and Sciences for the Academic Year 1896-1897, p. 34.

_____________________

Course Enrollment

[Economics] 4. Dr. John Cummings. — Theory and Methods of Statistics. — Applications to Economic and Social Questions. — Studies in the Movement of Population. 3 hours.

Total 15: 8 Seniors, 7 Juniors.

Source: Harvard University. Report of the President of Harvard College, 1896-1897, p. 65.

Economics 4.
Mid-Year Examination,
1896-97

[Divide your time equally between A. and B.]

A.

  1. The development of scientific statistics and the statistical method as employed in the social sciences.
  2. Social and economic causes of the migratory movements which have taken place in the populations of Europe and America during this century, and the laws in accordance with which those migrations have taken place where you can formulate any.

B.

(Take five.)

  1. Rural depopulation and the growth of cities in the United States.
  2. Define: “mean after life,” “expectation of life,” “mean duration of life,” “mean age at death.” What relation does the mean age of those living bear to the mean age at death? To the mean duration of life?
  3. Anthropological tests of race vitality as applied to the American negro?
  4. Explain how the economic value of a population is effected by its age and sex distribution.
  5. The United States census: either (1) an historical account of it, or (2) an account of the work now undertaken by the Census Bureau.
  6. Explain the various methods of calculating the birth rate of a population.
  7. How far are social conditions in a community revealed in the birth rate, the death rate, the marriage rate? Of what are fluctuations in these rates evidence in each case?
  8. What do you understand by the “index of mortality”?

Source: Harvard University Archives. Harvard University Mid-year Examinations, 1852-1943. Box 4. Bound volume: Examination Papers, Mid-Years. 1896-97.

Economics 4.
Year-End Examination,
1896-97

I.

  1. Give an historical account of the United States census, and a general statement of the ground covered in the census of 1890; also show how the census taking is supplemented by work done in the Department of Labor and in the statistical bureaus established in connection with the several administrative departments.
  2. Define Körösi’s “rate of natality,” and state any statistical evidence you know that the rate is affected by the standard of living.
  3. “It must, at all times, be a matter of great interest and utility to ascertain the means by which any community has attained to eminence among nations. To inquire into the progress of circumstances which have given pre-eminence to one’s own country would almost seem to be a duty….The task here pointed out has usually been left to be executed by the historian.” Porter: “The Progress of the Nation.”
    What contribution has statistics to make in the execution of this task? What do you understand to be the nature of the statistical method, and what are the legitimate objects of statistical inquiry?

II.

[Take two.]

  1. What light does statistics throw upon the “natural history of the criminal man”?
    Give Ferri’s classification of the “natural causes” of crime, and comment upon that classification. Of criminals.
    What do you understand by “rate of criminality”? By “criminal saturation”?
  2. To what extent in your opinion is suicide an evidence of degeneration in the family stock?
    Discuss the influence upon the rate of suicide of education, religious creed, race, climate and other facts of physical, political and social environment.
  3. Comment critically upon the tables relating to crime in the last five federal censuses taken in the United States.
  4. What difficulties beset a comparative study of criminality in different countries?
  5. How far is it possible to give a quantitative statement to moral and social facts?

III.

[Take one.]

  1. What are some of the more salient facts concerning the movement of population and wealth in the United States, England, and France during the present century, so far as those facts are evidenced in the production, consumption and distribution of wealth?
  2. Discuss the movement of wages and prices in the United States since 1890.
  3. What do you understand by “index figures,” “average wages,” “average prices,” and “weighted averages”?

IV.

[Take one.]

  1. How do you account for the increase in the proportion of urban to rural population during this century? What statistical evidence is there that the increased density of a population affects the mean duration of life? What importance to you attach to this evidence?
    Explain the effect of migratory movements upon the distribution of a population according to age, sex and conjugal condition, and upon the birth rate, death rate and marriage rate.
  2. Define and distinguish: “mean age at death”; “mean duration of life”; “mean age of those living”; “expectation of life.”
  3. The “law of population” as formulated by Malthus and by subsequent writers.

Source: Harvard University Archives. Harvard University, Examination papers, 1873-1915. Box 4, Bound volume: Examination Papers 1896-97. Papers set for Final Examinations in History, Government, Economics, Philosophy, Fine Arts, Architecture, and Music in Harvard College. June 1897, pp. 39-41.
Previously transcribed: https://www.irwincollier.com/harvard-semester-exams-for-statistics-john-cummings-1896-1900/

_____________________

Economics 51.
Railway Transportation
1896-97

Course Announcement

[Economics] 51 hf. Railway Transportation. — Lectures and written work. Half-course. Tu., Th., Sat., at 1.30 (first half-year). Professor Taussig.

Source: Harvard University. Announcement of the Courses of Instruction Provided by the Faculty of Arts and Sciences for the Academic Year 1896-1897, p. 34.

Course Enrollment

[Economics] 51. Mr. Virtue. — Railway Transportation. 3 hours.

Total 62: 2 Graduates, 33 Seniors, 20 Juniors, 5 Sophomores, 2 Others.

Source:   Harvard University, Annual Reports of the President and Treasurer of Harvard College, 1896-97, p. 65.

Economics 51.
Final Examination,
1896-97

[Omit one.]

  1. Sketch the railway history of Italy.
  2. What light does the internal improvement movement in the United States throw upon the question of the public management of the railway industry?
  3. What is the present position of the courts with regard to the power of State legislatures to fix railway rates? On what ground does the public claim the right to interfere in the fixing of such rates.
  4. Discuss the attempt made by any one of the State governments to control railways by means of a commission.
  5. Describe the Hungarian system of passenger tariffs and its effects.
  6. As a basis of rate making, what is the relative importance of the principle of “charging what the traffic will bear,” and that of charging according to the “cost of service”?
  7. It is said of the railway business that “where combination is possible, competition is impossible.” Why is this true? Is it peculiarly true in the railway business? Give your reasons fully.
  8. Discuss the rate policy set forth in the following passages:
    “In the proposed reform of our transportation taxes it will probably be found advisable, at the beginning, to follow the example of Sir Rowland Hill in his reform of the old English postal system, and adopt, as the uniform rate for each class of service for all distances, the lowest rate now charged for the shortest distance for that class of service.”… “If distances of hundreds of miles, can be safely disregarded in the local transportation of milk and potatoes and grain, then surely there is every reason to believe that a general grouping of all the railway stations in the country with a uniform rate will prove to be the best possible system that can be devised for the common good of all.”
  9. A railroad company subject to the “Interstate Commerce Law” charges a much lower rate from New Orleans to San Francisco, for goods which have been imported than for like goods of domestic manufacture. What is the decision of the courts as to the legality of such a charge? What is the economic justification for the decision?
  10. What do you regard as the greatest defects of the “Interstate Commerce Law,” as at present interpreted, and what legislation should you suggest to remedy such defects?
  11. What is the “railway problem”?

Sources: Harvard University Archives. Harvard University Mid-Year Examination, 1852-1943. Box 4. Bound volume: Examination Papers, Mid-Years 1896-97. Copy also in Harvard University Archives. Harvard University, Examination papers, 1873-1915. Box 4, Bound volume: Examination Papers 1896-97. Papers set for Final Examinations in History, Government, Economics, Philosophy, Fine Arts, Architecture, and Music in Harvard College. June 1897, pp. 41-42.

_____________________

Economics 61
History of Tariff Legislation in the United States,
1896-97

Course Announcement

[Economics] 62 [sic] hf. History of Tariff Legislation in the United States. Half-course. Tu., Th., at 2.30, and a third hour at the pleasure of the instructor (second [sic] half-year). Professor Taussig.

Source: Harvard University. Announcement of the Courses of Instruction Provided by the Faculty of Arts and Sciences for the Academic Year 1896-1897, p. 34.

Course Enrollment

[Economics] 61. Professor Taussig. — History of Tariff Legislation in the United States. Hf. 2 hours. 1st half-year.

Total 74: 7 Graduates, 39 Seniors, 20 Juniors, 3 Sophomores, 5 Others.

Source:   Harvard University, Annual Reports of the President and Treasurer of Harvard College, 1896-97, p. 66.

Economics 61.
Final Examination,
1896-97

[Arrange your answers strictly in the order of the questions. Answer all the questions.]

  1. How was the course of commercial history between 1789 and 1810 connected with customs legislation and with the state of opinion on the tariff during the same period?
  2. What were the “abominations” of the tariff act of 1828? What became of them?
  3. Was there similarity between the stages in tariff policy reached by England and by the United States in 1846?
  4. “The ultimate reduction of the price of American to that of the British rolled iron can only, and ultimately will, be accomplished in that western region, which abounds with ore, and in which is found the most extensive formation of bituminous coal that has yet been discovered in any part of the globe.” — Gallatin, in 1832.
    What were the conditions as to the production of iron in the United States when Gallatin wrote? When and how was his prediction fulfilled?
  5. When did the argument appear that protective duties serve to maintain a high rate of wages in the United States? Why at that time? How far do you think it sound?
  6. Why is no carpet wool raised in the United States? Why is no flax cultivated for fibre? Are high wages an obstacle to the production of such commodities?
  7. Was Webster a consistent advocate of free trade in 1824? Gallatin in 1832? Secretary Walker in 1846?
  8. Sketch the main features in the history of duties on wool and woollens from 1846 to 1896.
  9. Wool, pig iron, sugar, — on which would you now remit duties first, and on which last? Why? What are the present duties?

Source: Harvard University Archives. Harvard University Mid-Year Examination, 1852-1943. Box 4. Bound volume: Examination Papers, Mid-Years 1896-97.

Copy also in Harvard University, Examination papers, 1873-1915. Box 4, Bound volume: Examination Papers 1896-97. Papers set for Final Examinations in History, Government, Economics, Philosophy, Fine Arts, Architecture, and Music in Harvard College. June 1897, pp. 42-43.

_____________________

ECONOMICS 81.
History of Financial Legislation in the United States

[81 hf. History of Financial Legislation in the United States. Half-course. Tu., Th., at 2.30, and a third hour at the pleasure of the instructor (first half-year). Professor Dunbar.]

Omitted in 1896-97.

Source: Harvard University. Announcement of the Courses of Instruction Provided by the Faculty of Arts and Sciences for the Academic Year 1896-1897, p. 34.

_____________________

ECONOMICS 162.
Selected Topics in the Financial History of the United States,
1896-97

Course Announcement

[Economics] *161 [sic] hf. Selected Topics in the Financial History of the United States. Half-course. Tu., Th., at 2.30(first half-year [sic]). Professor Dunbar.
Course 16 may be taken as a full course by Graduate Students and by candidates for Honors in Political Science.

Source: Harvard University. Announcement of the Courses of Instruction Provided by the Faculty of Arts and Sciences for the Academic Year 1896-1897, p. 34.

Course Enrollment

[Economics] 62. Professor Dunbar. — Selected Topics in the Financial History of the United States. Hf. 2 hours. 2dhalf-year.

Total 21: 11 Graduates, 6 Seniors, 4 Juniors.

Source:   Harvard University, Annual Reports of the President and Treasurer of Harvard College, 1896-97, p. 66.

Economics 162.
Final Examination,
1896-97

  1. Under the present arrangement of our financial system how far is the original theory of the Independent Treasury Act of 1846 preserved?
  2. What are the considerations for and against the use of banks as government depositaries?
  3. Trace the steps by which the idea that redeemed notes might be reissued became familiar and was finally embodied in the existing law.
  4. How far was the successful resumption in 1879 the result of fortunate circumstances, not to be foreseen in 1875?
  5. What is the legal authority for receiving United States notes in payment of duties on imports?
  6. What would be the probable effect on the Treasury, if Congress were now to adopt a tariff producing ample revenue, without making any change in the currency legislation?
  7. In Secretary Sherman’s Report for 1879 “it is respectfully recommended that by law the resumption-fund be specially defined and set apart for the redemption of United States notes, and that the notes redeemed shall only be issued in exchange for or purchase of coin or bullion.”
    Trace the effect of such a provision in case of large exports of gold. How far would it have served to prevent the difficulties which have actually occurred since 1893?
  8. In his Report for 1889, Secretary Windom recommended the following measure: —
    “Issue Treasury notes against deposits of silver bullion at the market price of silver when deposited, payable on demand in such quantities of silver bullion as will equal in value, at the date of presentation, the number of dollars expressed on the face of the notes at the market price of silver, or in gold, at the option of the Government; or in silver dollars at the option of the holder. Repeal the compulsory feature of the present coinage act.”
    What was this measure expected to accomplish? How would its operation probably have differed from that of the Silver Purchase Act of 1890?

Source: Harvard University Archives. Harvard University Examination Papers, 1873-1915. Box 4. Bound volume: Examination Papers 1896-97. Papers Set for Final Examinations in Philosophy, History, Government, Economics, Fine Arts, Architecture, and Music in Harvard College, June 1897, pp. 51-52.

____________________

Economics 72.
The Theory and Methods of Taxation,
1896-97

Course Announcement

[Economics] *72 hf. The Theory and Methods of Taxation, with special reference to Local Taxation in the United States. Half-course. Mon., Wed., Fri., at 9 (second half-year). Mr. ——.

Source: Harvard University. Announcement of the Courses of Instruction Provided by the Faculty of Arts and Sciences for the Academic Year 1896-1897, p. 35.

Course Enrollment

[Economics] 72. Professor Taussig. — The Theory and Methods of Taxation, with special reference to Local Taxation in the United States. Hf. 3 hours. 2d half-year.

Total 51: 6 Graduates, 17 Seniors, 18 Juniors, 5 Sophomores, 5 Others.

Source:   Harvard University, Annual Reports of the President and Treasurer of Harvard College, 1896-97, p. 66.

Economics 72.
Final Examination,
1896-97

[Arrange your answers strictly in the order of the questions.]

  1. Which among the following would you call a “tax,” and why,—
    (a) an assessment for betterment (e.g. for a sidewalk) on real estate;
    (b) a liquor license;
    (c) the charge for a postage stamp;
    (d) the charge for tobacco in France;
    (e) the charge for a ticket on a Prussian railway.
  1. How far is there separation of local taxes on real estate from state taxes on real estate in France? in Prussia? in England? In American states? Which adjustment seems to you the best, and why?
  2. State the points of resemblance and the points of difference between the system of local taxation in England and the usual method of local taxation in the United States.
  3. Is a tax like the French personelle-mobilière adapted for use in American states? one like the Prussian Business Tax?
  4. Do you think progressive taxation to be sound in principle? Why? or why not?
  5. Point out similarities and differences between the methods of taxing the holders of securities in England and in Pennsylvania.
  6. What do you conceive to be the methods and effects of the taxation of mortgage debts in Massachusetts? in California?
  7. On what grounds would you approve or disapprove of taxes on inheritances and successions, levied by the several American states? of income taxes, similarly levied?
  8. “The statute in Massachusetts, which taxes corporations on their capital stock less the value of real estate and machinery, is indefensible. According to the Massachusetts law, corporations are taxable locally only on their real estate and machinery, while they are taxable for commonwealth purposes only on the value of their capital stock deducting the value of the machinery and the real estate, they are therefore taxed only once on their total property. Individuals, on the other hand, pay not only a general tax for state purposes, but also another general property tax for local purposes. Corporations thus are treated more leniently than individuals.” — Seligman.
    Is this an accurate statement of the legislation in Massachusetts? and are corporations more leniently dealt with than individuals?

One of the following questions may be substituted for any one of the preceding.

  1. What distinctions are made, in the process of assessment under the Prussian income tax, between incomes under 3000 marks and those above?
  2. How are state and local control combined in the assessment of income taxes in Prussia? in England?

Source: Harvard University Archives. Harvard University, Examination papers, 1873-1915. Box 4, Bound volume: Examination Papers 1896-97. Papers set for Final Examinations in History, Government, Economics, Philosophy, Fine Arts, Architecture, and Music in Harvard College. June 1897, pp. 43-44.

_____________________

Economics 71.
Financial Administration and Public Debts

[[Economics] *71 hf. Financial Administration and Public Debts. Half-course. Tu., Th., Sat., at 11 (first half-year). Professor Dunbar.]

Omitted in 1896-97. Courses 71 and 72 are usually given in alternation with Courses 121 and 122.

Source: Harvard University. Announcement of the Courses of Instruction Provided by the Faculty of Arts and Sciences for the Academic Year 1896-1897, p. 35.

_____________________

Economics 121.
Banking and the History of the Leading Banking Systems

Course Announcement

[Economics] *121 hf. Banking and the History of the leading Banking Systems. Half-course. Tu., Th., Sat., at 11 (first half-year). Professor Dunbar.

Source: Harvard University. Announcement of the Courses of Instruction Provided by the Faculty of Arts and Sciences for the Academic Year 1896-1897, p. 35.

Course Enrollment

[Economics] 121. Professor Dunbar. — Banking and the History of the leading Banking Systems. Hf. 3 hours. 1st half-year.

Total 47: 1 Graduate, 18 Seniors, 24 Juniors, 4 Others.

Source:   Harvard University, Annual Reports of the President and Treasurer of Harvard College, 1896-97, p. 66.

Economics 121
Final Examination,
1896-97

  1. If, as McLeod says, all modern banks are banks of issue, how is it that discussions and legislation about banking are chiefly taken up with questions as to bank notes?
  2. When can a note currency be said to be elastic, and what is necessary to give it that quality? Illustrate by actual cases.
  3. Name as many banking systems as you can, which
    (a) protect creditors by a stockholders’ liability of any sort; or
    (b) protect notes by a prior lien on assets.
  4. What advantages are derived from the system of branch banks, and where is it used?
  5. Which of the great banks, — the Bank of England, the Bank of France, the Reichsbank, —  appears to have the best title to be called a government bank?
  6. Suppose that, in a country having well developed banks, a large issue of inconvertible legal tender notes should be made; in what way should you expect its inflating effect upon prices and credits to be produced?
  7. How is the unequal distribution of national banks in the United States, during the years when banking under the national act was most profitable, to be explained?
  8. The following items being given, viz. :
Public Deposits £7.7 Other Securities £30.0
Other Deposits £48.0 Notes in circulation £28.1
Government Securities £20.7 Coin and Bullion £40.4
Government Debts £11.0

construct a Bank of England account, with its separate Departments of Issue and Banking.

  1. Why is it that a comparison of the English country bank circulation with the Scotch shows that one is gaining while the other is dying out?
  2. Under the German bank act what are the two requirements as to holding cash? What is counted as cash under these requirements respectively? Under what conditions does a bank find one or the other of these requirements practically inoperative?
  3. The account of the Bank of France may be simplified as follows:—
Cash, Surplus, and Profits fr. 225. Loans and Investments fr. 1.141.
Sundries fr. 72. Government Securities fr. 353.
Deposits fr. 822. Sundries fr. 75.
Notes fr. 3.612 Cash fr. 3.162.
fr. 4.731. fr. 4.731.

How much of its circulation could the bank pay off and yet earn its present profit? State the account as it would appear if such a change were made?

Source: Harvard University Archives. Harvard University, Examination papers, 1873-1915. Box 4, Bound volume: Examination Papers 1896-97. Papers set for Final Examinations in History, Government, Economics, Philosophy, Fine Arts, Architecture, and Music in Harvard College. June 1897, pp. 47-48.

_____________________

Economics 122.
International Payments and the Flow of the Precious Metals,
1896-97

Course Announcement

[Economics] *122 hf. International Payments and the Flow of the Precious Metals. Half-course. Tu., Th., Sat., at 11(second half-year). Professor Dunbar and Mr. Meyer.
Courses 121 and 122 are usually given in alternation with Courses 71 and 72.

Source: Harvard University. Announcement of the Courses of Instruction Provided by the Faculty of Arts and Sciences for the Academic Year 1896-1897, p. 35.

Course Enrollment

[Economics] 122. Professor Dunbar and Mr. Meyer. — International Payments and the Flow of the Precious Metals. Hf. 3 hours. 2d half-year.

Total 20: 9  Graduates, 2 Seniors, 6 Juniors, 3 Others.

Source:   Harvard University, Annual Reports of the President and Treasurer of Harvard College, 1896-97, p. 66.

Economics 122.
Final Examination,
1896-97

It is expected that one-half of the time of this examination will be required for the questions in division A of this paper.

A.

  1. It is often said that the resumption of specie payments by France and by the United States and the adoption of the gold standard by Germany made nearly simultaneous demands upon the world’s stock of gold. Discuss this statement at length.
  2. The transfer of international securities,
    (a) during the funding operations of the United States, 1872-79;
    (b) as a consequence of the French Indemnity payments.
    Take one of these two cases.
  3. The conditions which led to the flow of gold to the United States,
    (a) in 1896;
    (b) in the fiscal years 1880 and 1881.
    Take one of these two cases.

B.

  1. What were the contributions of Ricardo, Mill and Cairnes, respectively, to the discussion of the laws determining the exchange of commodities between nations?
  2. To what extent are the principles involved in exchanges of goods and services between nations also applicable in domestic exchanges?
  3. Describe the” triangular” operation in exchange between three countries, whereby an export of specie may take place from one of them before the price of exchange has fallen to the shipping point. Illustrate by an actual case.
  4. Explain the difference in rates for long exchange as compared with short, and show the conditions under which an unusual divergence of rates may exist. Illustrate by an actual case.
  5. Why is it that in the dealings between England and other countries bills of exchange are chiefly drawn upon England and few are drawn by her upon others? How are the transactions between England and the United States adjusted, when the bills are for the most part drawn by us upon England?

Source: Harvard University Archives. Harvard University, Examination papers, 1873-1915. Box 4, Bound volume: Examination Papers 1896-97. Papers set for Final Examinations in History, Government, Economics, Philosophy, Fine Arts, Architecture, and Music in Harvard College. June 1897, pp. 48-49.

_____________________

 

Primarily for Graduates.

Economics Seminary Announcement

[Economics] 20. Seminary in Economics. Mon., at 4.30.
Professors Dunbar, Taussig, and Ashley and Asst. Professor Edward Cummings will guide competent students in research on topics assigned after consultation. The Seminary will hold weekly meetings; and in addition each student will confer once a week, with the instructor under whose guidance he carries on his investigations.

Source: Harvard University. Announcement of the Courses of Instruction Provided by the Faculty of Arts and Sciences for the Academic Year 1896-1897, p. 35.

Economics Seminary Enrollment

[Economics] 20. Professors Dunbar, Taussig, and Ashley and Asst. Professor Edward Cummings. — Seminary in Economics.

Total 20: 17 Graduates, 2 Seniors, 1 Other.

Source:   Harvard University, Annual Reports of the President and Treasurer of Harvard College, 1896-97, p. 66.

Image Source: Memorial Hall, Harvard University. From Library of Congress Prints and Photographs Division Washington, D.C.

 

 

 

 

Categories
Exam Questions Harvard

Harvard. Mid-year and Year-End Final Exams in Economics and Social Ethics, 1894-1895

 

 

With this post Economics in the Rear-view Mirror adds yet another annual collection of final examination questions for the economics courses offered at Harvard together with the questions from Professor Peabody’s “Ethics of the Social Questions” that covered issues such as poverty, labor relations, and socialism (as opposed to doctrines of individualism). In 1894-95 Frank Taussig was on sabbatical leave in Italy which accounts for his whereabouts that academic year.  Today I learned that “doctrine” was understood as a synonym for “theory” during the gay nineties, see Economics 2 (Economic Theory from Adam Smith to the present time) below.

Exams for one course taught were not included in the published collection of exams. It was Edward Cummings course Economics 14 (Philosophy and Political Economy.—Utopian Literature from Plato’s Republic to the present time). Exams for Economics 14 given in other years have been transcribed and posted.

__________________

1894-95.
PHILOSOPHY 5.

Course Title, Staffing, and Enrollment

[PHILOSOPHY] 5. Professor [Francis G.] Peabody. — The Ethics of the Social Questions. — The questions of Charity, the Family, Temperance, and the various phases of the Labor Question, as problems of practical Ethics. — Lectures, essays, and practical observations. 2 hours.

Total 84: 1 Graduate, 40 Seniors, 15 Juniors, 3 Sophomores, 25 Others.

Source: Harvard University. Report of the President of Harvard College, 1894-1895, p. 59.

 

PHILOSOPHY 5.
THE ETHICS OF THE SOCIAL QUESTIONS
Mid-Year Examination. 1895.

[Omit one question.]

  1. The Ethical Idealism of Plato, of Aristotle, and of Kant, compared with the modern doctrine of duty.
  2. Professor Sumner’s doctrine of the Social Fulcrum vs. the philosophy of scientific charity.
  3. Indicate, very briefly, the place in the History of Philanthropy of:

Frédéric Le Play,
Dorothea Dix,
Pastor von Bodelschwingh,
Charles L. Brace,
Samuel G. Howe.

  1. The Elberfeld System — its organization, officials, relation tomunicipal government, and practical working.
  2. The Liverpool System of Collection.
  3. Mr. Charles Booth’s eight classes of East London,— their definition, dimensions, traits, and proportion. (Labor and Life of the People, I. pp. 37-62.) Mr. Booth’s view of the children of Class E (p. 160).
  4. Compare Mr. Booth’s method and results in East London with his method and results in all London.
  5. Compare the principle as to direct relief of the London Charity Organization Society with that of the Boston Associated Charities. (Loch, Charity Organization, pp. 59, 82.) Which is the sounder principle? Why?
  6. The Belgian Labor Colonies,— their scope and method of classification. Compare their aims with those of the colonies of Holland and Germany.
  7. The Christian doctrine of the Social Order — its principles and its peril.

Source: Harvard University Archives. Harvard University, Mid-year examinations, 1852-1943. Box 3. Bound volume Examination Papers, Mid-Year 1894-95.

 

PHILOSOPHY 5.
THE ETHICS OF THE SOCIAL QUESTIONS.
Year-end Examination. 1895.

  1. Explain the theory of ethics which makes the basis of this course of study; and the way in which this theory is practically illustrated by phases of the modern labor question.
  2. In what respect do the social ideals of Carlyle and Ruskin seem identical, and in what respect do they appear to be inconsistent with each other?
  3. The authorship and the significance of the following phrases:

“There is no wealth but Life…. A strange political economy; the only one, nevertheless, that ever was or can be.”

“I am for permanence in all things. Blessed is he that continueth where he is.”

“The gospel of dilettantism.”

“Roots of honour.”

“Ricardo is the parent of Socialism.”

“The value of a thing is independent of opinion and of quantity.”

“The reformation was the work of a monk; the revolution must be the work of a philosopher.”

“The people are the Rock on which the Church of the future must be built.”

  1. The practical programme proposed by Scientific Socialism; the chief advantages claimed for it by its adherents; and the criticisms on it which appear to you most serious. Utilize here your reading of Naquet and The Social Horizon.
  2. Socialism and Religion. The apparent grounds for sympathy and the practical reasons for antagonism. The teachings concerning socialism in the Encyclical of 1891.
  3. The philosophy of history which encourages the Socialist, and the “Opportunist’s” view of this “Law” of social evolution.
  4. The growth of Trades Unionism in Great Britain, and its contribution to moral education.
  5. Federalism and Individualism in English Coöperation. The issue involved, and the advantages of each scheme of expansion.
  6. Compare the characteristics of the forms of Liquor Legislation in force in Massachusetts and in Pennsylvania. (Fanshawe, XI, XII.) How are licenses granted under the Brooks Law? What is the function of probation-officers in Massachusetts?
  7. How far do physiological considerations go to determine one’s duty as to drink?

Source: Harvard University Archives. Harvard University, Examination Papers, 1873-1915. Box 4. Bound volume: Examination Papers, 1893-95. “Papers Set for Final Examinations in Philosophy, History, Government and Law, Economics Fine Arts, Architecture, and Music in Harvard College, June, 1895,” pp. 6-7.

__________________

1894-95.
ECONOMICS 1.

Course Title, Staffing, and Enrollment

[ECONOMICS] 1. Professor [William] Ashley, Asst. Professor [Edward] Cummings, Dr. [John] Cummings, and Mr. [Frederick Redman] Clow. — Outlines of Economics. — Mill’s Principles of Political Economy. — Lectures on Economic Development, Distribution, Social Questions, and Financial Legislation. 3 hours.

Total 277: 2 Graduates, 39 Seniors, 18 Juniors, 159 Sophomores, 9 Freshmen, 50 Others.

Source: Harvard University. Report of the President of Harvard College, 1894-1895, p. 62.

 

ECONOMICS 1.
Mid-Year Examination. 1895.

[Arrange your answers strictly in the order of the question. One question may be omitted.]

  1. “All members of the community are not laborers, but all are consumers, and consume either unproductively or productively.” Explain and illustrate by examples. Suppose everybody resolved to consume “productively” only, what would be the result?
  2. “The distinction, then, between capital and not-capital, does not lie in the kind of commodities, but in the mind of the capitalist — in his will to employ them for one purpose rather than another.” Discuss this statement, using the following illustrations:—

Bread.
A knitting machine.
A steam engine.
A carriage.

  1. Where does true economic rent appear in the following cases:—

(a) The cultivation of a farm by its owner.
(b) The rental of a farm under a long lease by a tenant who has made permanent improvements on the land.

  1. What is the effect on values of a general fall of profits? Of a general fall of wages?
  2. What is the effect on rents of (1) an improvement in the methods of agriculture, (2) an improvement in transportation?
  3. “The price of land, mines, and all other fixed sources of income, depends on the rate of interest.” Explain.
  4. According to Mill, “Every addition to capital gives to labor either additional employment, or additional remuneration.” Why? What is the effect of an increase of labor-saving machinery on employment and on remuneration? Illustrate carefully.
  5. “Money cannot in itself perform any part of the office of capital, since it can afford no assistance to production.” Do you agree or disagree? Why? Is money capital? Is credit money? Is credit capital?
  6. What does Mill mean by “stationary state”? And what changes would bring about a progressive state?
  7. What would be the effect on prices of (1) adding to a gold and silver currency a small issue of inconvertible paper money, (2) the discovery of very rich gold fields?
  8. What do you understand by the Domestic system? By Competition? By Labor?

Source: Harvard University Archives. Harvard University, Mid-year examinations, 1852-1943. Box 3. Bound volume Examination Papers, Mid-Year 1894-95.

 

ECONOMICS 1.
Year-end Examination. 1895.

(Arrange your answers strictly in the order of the question. Omit three of the even numbers: answer all others.)

  1. “We must suppose the entire savings of the community to be annually invested in really productive employment within the country itself; and no new channels opened by industrial inventions, or by more extensive substitution of the best known processes for inferior ones.” How would profits be affected supposing population (a) to remain stationary; (b) to increase in proportion to the increase in capital?
  2. The operations, therefore, of speculative dealers, are useful to the public whenever profitable to themselves; and although they are sometimes injurious to the public, by heightening the fluctuations which their more usual office is to alleviate, yet, whenever this happens the speculators are the greatest losers. Explain Mill’s reasoning.
  3. Mill says of the stationary state, “I am inclined to believe that it would be, on the whole, a very considerable improvement on our present condition.” Why? Explain carefully.
  4. Is there a necessary hostility of interests between consumers organized in co-operative associations and producers organized in trade unions?
  5. Describe the different results obtained in co-operation by distributing profits in the form of dividend (a) on capital, (b) on labor (in proportion to wages), (c) on purchases. Illustrate by the experience of co-operation in France and England.
  6. How do you distinguish between what Mill calls the necessary and the optional functions of government?
  7. “We have had an example of a tax on exports, that is, on foreigners, falling in part on ourselves. We shall therefore not be surprised if we find a tax on imports, that is, on ourselves, partly falling on foreigners.” Explain carefully each case, tracing the possible effects upon prices and international trade of taxes (a) upon exports; (b) upon imports.
  8. “Equality of taxation, therefore, as a maxim of polities, means equality of sacrifice.” Apply this maxim to a tax on incomes.
  9. Suppose a tax of a fixed sum per bushel to be laid upon corn; what would be the effect (a) upon prices; (b) upon population; (c) upon profits; (d) upon rents?
    How would the results differ if instead of a fixed sum per bushel the tax were…

(i) …a fixed proportion of the produce;
(ii) …proportioned to the rent of the land;
(iii) …a fixed sum of so much per cultivated acre? Explain carefully each case.

  1. Describe the kinds of currency used in the United States, indicating briefly the conditions of issue in each case.
  2. Explain the causes and effects of (a) combined reserves, (b) a suspension of the Bank Charter Act in England.
  3. What are the provisions of the law in regard to the issue of bank notes at the present day in England? In Germany?

Source: Harvard University Archives. Harvard University, Examination Papers, 1873-1915. Box 4. Bound volume: Examination Papers, 1893-95. “Papers Set for Final Examinations in Philosophy, History, Government and Law, Economics Fine Arts, Architecture, and Music in Harvard College, June, 1895,” pp. 33-34.

__________________

1894-95.
ECONOMICS 2.

Course Title, Staffing, and Enrollment

[ECONOMICS] 2. Professors Ashley and [Silus Marcus] Macvane. — Economic Theory from Adam Smith to the present time. — Selections from Adam Smith and Ricardo. — Modern Writers. — Lectures. 3 hours.

Total 34: 9 Graduates, 14 Seniors, 6 Juniors, 1 Sophomores, 4 Others.

Source: Harvard University. Report of the President of Harvard College, 1894-1895, p. 62.

 

ECONOMICS 2.
Mid-Year Examination. 1895.

N.B.—Not more than seven questions must be attempted.

  1. “The study which lately in England has been called Political Economy is, in reality, nothing more than the investigation of some accidental phenomena of modern commercial operations, nor has it been true in its investigation even of these. It has no connection whatever with political economy, as understood and treated of by the great thinkers of past ages; and as long as its unscholarly and undefined statements are allowed to pass under the same name, every word written on the subject by those thinkers—and chiefly the words of Plato, Xenophon, Cicero, and Bacon—must be nearly useless to mankind” (Ruskin, Munera Pulveris). Consider some or all of these assertions.
  2. Give a brief account of the Physiocrat doctrine, and state to what extent it was “corrected” by Adam Smith.
  3. Explain the origin and content of Adam Smith’s conception of “Nature.”
  4. “A diamond has scarcely any value in use.” Consider this statement in its relation to the discussion since Adam Smith’s time of the doctrine of Value.
  5. How does the doctrine of Rent expounded by Adam Smith agree with, and differ from, that of Ricardo?
  6. Compare Adam Smith’s “natural rate of wages” with Ricardo’s “natural price of labour.”
  7. “Population tends to outstrip the means of subsistence.” Distinguish the various meanings assignable to this phrase, and indicate which was meant by Malthus.
  8. What does Adam Smith understand by “Capital”? Compare his conception with that of John Stuart Mill.
  9. Present a critical estimate—based upon your own study—of one of the following:

1. Ingram, History of Political Economy.
2. Price, Political Economy in England.
3. Cossa, Introduction to the Study of Political Economy.

Source: Harvard University Archives. Harvard University, Mid-year examinations, 1852-1943. Box 3. Bound volume Examination Papers, Mid-Year 1894-95.

 

ECONOMICS 2.
Year-end Examination, 1895

Answer at least four, but not more than six, of the following questions:

  1. What is the economic source of Interest? Examine the proposition that “interest is the price paid for the use of capital.”
  2. State briefly your conclusions as to the law of general wages.
  3. Apply the Austrian theory of wages to the following case:
    Number of laborers 1,000,000; total subsistence fund $600,000,000; scale of increase of productiveness of labor as the “productive period” is lengthened from one year to seven years: $350, $450, $530, $580, $620, $650, and $670.
  4. How, in your opinion, are the profits of employers determined? What is your conclusion as to the function, in distribution, of the so-called “no profits employers.”
  5. Discuss the following passages:
    “This National Dividend is at once the aggregate Net product of, and the sole source of payment for, all the agents of production within the country: it is divided up into Earnings of labour, Interest of capital, and lastly the Producer’s Surplus, or Rent, of land and of other differential advantages for production. It constitutes the whole of them and the whole of it is distributed among them.”
    “The proposal to put rent aside while we are considering how earnings and interest are determined, has been found to suggest that rent is determined first and then takes part in determining earnings and interest; and this is, of course, the opposite of what really occurs.”
  6. It has been said that Mill expresses his meaning badly when he said that demand for commodities is not a demand for labor. Does the proposition seem to you to need revision!
  7. Does increase of saving tend to make the supply of goods outrun the demand for goods?
  8. Examine the doctrine that the exchange value of commodities is determined by marginal utility.
  9. Past and present relations between gold and silver.

Source: Harvard University Archives. Harvard University, Examination Papers, 1873-1915. Box 4. Bound volume: Examination Papers, 1893-95. “Papers Set for Final Examinations in Philosophy, History, Government and Law, Economics Fine Arts, Architecture, and Music in Harvard College, June, 1895,” p. 35.

__________________

1894-95.
ECONOMICS 3.

Course Title, Staffing, and Enrollment

[ECONOMICS] 3. Asst. Professor Cummings. — The Principles of Sociology. — Development of the Modern State, and of its Social Functions. 2 hours.

Total 52: 10 Graduates, 30 Seniors, 4 Juniors, 3 Sophomores, 5 Others.

Source: Harvard University. Report of the President of Harvard College, 1894-1895, p. 62.

 

ECONOMICS 3.
Mid-Year Examination. 1895.

Answer the questions in the order in which they stand. Omit three questions.

  1. State accurately the reading you have done in this course to date.
  2. “But now let us drop the alleged parallelism between individual organizations and social organizations. I have used the analogies elaborated but as a scaffolding to help in building up a coherent body of sociological inductions. Let us take away the scaffolding: the inductions will stand by themselves.” What are these inductions?
  3. “The family relinquishes one provisional and temporary function after another; its only purpose being to fill gaps in social offices, it made way for independent institutions … as soon as these institutions arose.” Explain and illustrate. How far would Spencer assent to this doctrine?
  4. “Most anthropologists who have written on prehistoric customs believe, indeed, that man lived originally in a state of promiscuity or ‘communal marriage’; but we have found this hypothesis is essentially unscientific.” Discuss the evidence.
  5. “The status of children, in common with that of women, rises in proportion as the compulsory coöperation characterizing militant societies, becomes qualified by the voluntary coöperation characterising industrial societies.” Why? Trace the rise, and illustrate.
  6. “These three distinct states of mind, all of which, in point of fact, are admitted to exist together at the present time, and perhaps to have always done so to a greater or less extent, Comte declares to have undergone a regular progressive movement in the history of society. There have been three successive epochs, during which these philosophic principles, each in its turn, preponderated over both the others and controlled the current of human events.” Explain.
  7. “So that as law differentiates from personal commands, and as morality differentiates from religious injunctions, so politeness differentiates from ceremonial observance. To which I may add, so does rational usage differentiate from fashion.” Explain and illustrate.
  8. How does Spencer account for the diverse types of political organization; and what influences determine the order in which they arise? Illustrate.
  9. “From the Evolution-standpoint we are thus enabled to discern the relative beneficence of institutions which, considered absolutely, are not beneficent; and are taught to approve as temporary that which, as permanent, we abhor.” Explain and illustrate. Does our idea of progress then include all social changes?
  10. “In all ways, then, we are shown that with this relative decrease of militancy and relative increase of industrialism, there has been a change from a social order in which individuals exist for the State, to a social order in which the State exists for individuals?” Explain and illustrate.
  11. According to Spencer, what are likely to be the future forms of political organization and action in societies that are favorably circumstanced for carrying social evolution to its highest stage?
  12. “At bottom this is a physical explanation, and Spencerian sociology in general, whether formulated by Mr. Spencer or by other writers under the influence of his thought, is essentially a physical philosophy of society, notwithstanding its liberal use of biological and psychological data.” Do you agree or disagree? Why?

Source: Harvard University Archives. Harvard University, Mid-year examinations, 1852-1943. Box 3. Bound volume Examination Papers, Mid-Year 1894-95.

 

ECONOMICS 3.
Year-end Examination

[Answer the questions in the order in which they stand. Omit one question.]

  1. State accurately the reading you have done in this course to date.
  2. What has been the function of religion in social evolution? (Compare Spencer and Kidd.) Do you find reasons for thinking society will become more religious?
  3. “The only conclusion to which we are brought by this prolonged examination of authorities is that community of land has not yet been historically proved.” Discuss the evidence.
  4. “And as of old, Society and State tend to coincide, political questions to become identical with social questions.” Discuss the historical changes and tendencies in question. Distinguish carefully between Society, the State, the Government, the Nation.
  5. “It is becoming clear that, when people speak of natural rights of liberty, property, etc., they really mean, not rights which once existed and have been lost, but rights which they believe ought to exist, and which would be produced by a condition of society and an ordering of the State such as they think desirable.” Explain. How far do changes in the theory and practice of penal legislation substantiate this view?
  6. “The gulf between the state of society towards which it is the tendency of the process of evolution now in progress to carry us, and socialism, is wide and deep.”
    “The Individualism of the past is buried, and the immediate future is unmistakably with a progressive Socialism, the full extent of which no man can get see.” Discuss carefully the facts and theories upon which these opposing views are based.
  7. “The philanthropic and experimental forms of socialism, which played a conspicuous rôle before 1848, perished then in the wreck of the Revolution, and have never risen to life again.” What were the characteristics of these earlier forms; and what was their relation to the movements which preceded them and followed them?

Source: Harvard University Archives. Harvard University, Examination Papers, 1873-1915. Box 4. Bound volume: Examination Papers, 1893-95. “Papers Set for Final Examinations in Philosophy, History, Government and Law, Economics Fine Arts, Architecture, and Music in Harvard College, June, 1895,” pp. 35-36.

__________________

1894-95.
ECONOMICS 5.

Course Title, Staffing, and Enrollment

[ECONOMICS] 52. Mr. George Ole Virtue. — Railway Transportation. — Lectures and written work. 3 hours. 2d half-year.

Total 21: 2 Graduates, 10 Seniors, 6 Juniors, 1 Sophomores, 2 Others.

Source: Harvard University. Report of the President of Harvard College, 1894-1895, p. 62.

 

ECONOMICS 5.
Year-end Examination. 1895.

  1. Sketch the railroad history of France.
  2. “The [Reilly] bill now before Congress proposes to extend the debt for another fifty years and a grand opportunity will thus be let slip for trying, under the most favorable circumstances, an experiment whose possibilities no man can measure.”
  3. What legislation can you suggest for improving the relations between the different classes of owners of railway capital? For the protection of the interests of investors in railway capital generally?
  4. State briefly the significance in railway history of the following cases: Munn v. Illinois; Wabash, etc. Ry. Co. v. Illinois; Ames v. U. P. Ry. Co.; Budd v. New York; In re Louisville & Nashville; The Denaby Main Colliery Case.
  5. Choose one:

(a) The bearing upon the making of rates, of the “cost of service”; “value of service”; “charging what the traffic will bear”; “joint cost.”
(b) “Group rates,” “equal mileage rates,” “the blanket rate,” “the postage rate,” “Wagen-raum tarif,” “differentials.”
(c) A “reasonable rates.”

  1. Recount the experience which has led the Interstate Commerce Commission to recomment an amendment to the Act to Regulate Commerce: (a) Construing the meaning of “the word ‘line’ when used in the act to be a physical line and not a business arrangement”; (b) relieving “shippers and individuals not connected with railway employment from liability to fine and imprisonment under Section 10,” with certain exceptions.
  2. What would be the probable effect of giving the Commission power to prescribe minimum as well as maximum rates? Would it obviate the necessity now claimed for pooling?
  3. “When the first bill to regulate commerce was passed the great and powerful wedge of State socialism, so far as government control of railroads is concerned, was driven one-quarter of its length into the timber of conservative government. … The pending bill, [the pooling which passed the House at the last session is referred to] the moment it becomes a law, will drive the wedge three-quarters of its length into the timber.”
    Give your reasons for agreeing or disagreeing with each of the above statements.
  4. What conclusions on the question of public management can you draw from the experience of the states in the internal improvement movement?
  5. Why is it peculiarly true in railway business that “competition must end in combination”?
  6. The success of the State Railroad Commissions and suggestions for increasing their efficiency.

Source: Harvard University Archives. Harvard University, Examination Papers, 1873-1915. Box 4. Bound volume: Examination Papers, 1893-95. “Papers Set for Final Examinations in Philosophy, History, Government and Law, Economics Fine Arts, Architecture, and Music in Harvard College, June, 1895,” pp. 36-37.

__________________

1894-95.
ECONOMICS 71.

Course Title, Staffing, and Enrollment

[ECONOMICS] 71. Professor [Charles F.] Dunbar. — The Theory and Methods of Taxation, with special reference to local taxation in the United States. 3 hours. 1st half-year.

Total 28: 6 Graduates, 11 Seniors, 9 Juniors, 2 Sophomores.

Source: Harvard University. Report of the President of Harvard College, 1894-1895, p. 62.

 

ECONOMICS 71.
Mid-year Examination. 1895.

  1. What is the “Benefit Theory” of taxation? What is the “Faculty Theory”? Define “Faculty” as used in this expression.
  2. What are the leading points of difference between the English, Prussian and American income tax systems?
  3. What reasons are there for having income tax levied by national authority rather than local? To what extent, if at all, do these reasons apply also to a general property tax?
  4. In levying a general property tax, should the debts of the taxpayer be deducted from the property held by him?
  5. By what reasoning is it maintained that,—
    “When the local real estate tax is levied according to rental value and assessed in the first instance on the occupier, as is the case in England, the main burden of the tax will rest ultimately on the occupier, not the owner of the premises.”
    Will the same reasoning apply to the income tax on rent, assessed under Schedule A., and collected from the occupier?
  6. What are the leading points of difference between the German method of taxing distilled liquors and the method practised in England and the United States?
  7. The theories of Canard, Thiers and Stein are,—
    “That every tax is shifted on everybody — that every consumer will again shift the tax on a third party, and that this third party who is again a consumer will shift it to someone else — and so ad infinitum. And since everyone is a consumer, everyone will bear a portion of the taxes that everybody else pays.”
    Professor Seligman’s comment is that “the error of this doctrine lies in the failure to distinguish between productive and unproductive consumption.” Is this answer complete? If not, wherein does it fail?
  8. In a statement of the circumstances under which a tax may or may not be capitalized, it is said,—
    “The principle would not apply to special taxes on property or profits if the capital value of this class of commodities should for any other reason fluctuate in price. For example, if a special tax were levied on government securities it might nevertheless happen that if some reason confidence in government bonds, as over against general securities, might decrease to such an extent as to counterbalance the decreased returns from the investment. In such a case there would be no capitalization of the tax.”
    What criticism have you to make on this reasoning?
  9. Can the theory of progressive taxation be satisfied by a gradually decreasing rate of progression [“degressively progressive taxation”]. or does it require a rate which shall cut off all income or accumulation above a certain level?
  10. What practical difficulties does the taxation of real estate offer in shaping a system of progressive taxation?

Source: Harvard University Archives. Harvard University, Examination Papers, 1873-1915. Box 4. Bound volume: Examination Papers, 1893-95. “Papers Set for Final Examinations in Philosophy, History, Government and Law, Economics Fine Arts, Architecture, and Music in Harvard College, June, 1895,” pp. 37-38.

__________________

1894-95.
ECONOMICS 72.

Course Title, Staffing, and Enrollment

[ECONOMICS] 72. Professor Dunbar. — Financial Administration and Public Debts. 3 hours. 2d half-year.

Total 28: 7 Graduates, 11 Seniors, 9 Juniors, 1 Sophomores.

Source: Harvard University. Report of the President of Harvard College, 1894-1895, p. 62.

 

ECONOMICS 72.
Year-end Examination

[Spend an hour on A, and the remainder of the time on B.]

A.

  1. Give an account of the management of the English debt in the decade 1880-90.
  2. Do “sound rules of finance” demand that the principal of the debt or the rate of interest shall be determined by the government? that securities shall never be issued below par? that a government shall not buy in its securities at a premium?

B.

  1. How far, if at all, is the government justified in pledging itself to any fixed policy of debt payment?
    How may the policy of conversation conflict with the policy of debt payment?
  2. Give an account of the United States refunding operations in the decade 1865-75.
  3. Discuss the respective powers of the Secretary of the Treasury of the United States, the Chancellor of the Exchequer in Great Britain, the minister of finance in France.
    In each case where does the responsibility for the financial policy of the government rest?
  4. Give an account of the creation of Pit’s sinking fund and of the successive modifications made in the sinking fund provisions down to 1803.
  5. Discuss the various methods of placing government securities in the market, and the conditions of contract which make one form of security more attractive to buyers than another.
  6. The United States 4 per cent. 30-year bonds are quoted at about 123¼; how is the present worth of these securities determined?
    What determines the present worth of a terminable annuity? of a perpetual annuity? of a life annuity?
  7. Discuss the manner of making up the estimates of public income and expenditure in Great Britain and in France; the manner of providing for any deficits which may occur in any department during the year; the manner of providing for carrying on the government where the enactment of the budget is delayed until after the beginning of the year; and the disposal of balances unexpended at the end of the year.

Source: Harvard University Archives. Harvard University, Examination Papers, 1873-1915. Box 4. Bound volume: Examination Papers, 1893-95. “Papers Set for Final Examinations in Philosophy, History, Government and Law, Economics Fine Arts, Architecture, and Music in Harvard College, June, 1895,” p. 39.

__________________

1894-95.
ECONOMICS 8.

Course Title, Staffing, and Enrollment

[ECONOMICS] 81. Professor Dunbar. — History of Financial Legislation in the United States. 2 hours.

Total 52: 5 Graduates, 22 Seniors, 22 Juniors, 3 Others.

Source: Harvard University. Report of the President of Harvard College, 1894-1895, p. 62.

 

ECONOMICS 8.
Mid-Year Examination. 1895.

  1. Hamilton is sometimes said to have favored the policy of perpetual debt, and Gallatin, on the other hand, to have established the policy of debt-payment. How far are these statements confirmed by the measures of Hamilton and Gallatin respectively?
  2. How far should you say that Hamilton was justified in his expectation (stated in the Report on Public Credit), (1) That the public debt, if properly funded, would answer most of the purposes of money, and (2) that it would increase the amount of capital for use in trade and lower the interest of money?
  3. When were the several classes of obligations in which the revolutionary debt was funded finally paid off?
  4. Was it fortunate or unfortunate that Congress did not adopt Madison’s policy as to a United States Bank in January, 1815? Why?
  5. Give a list, with dates, of the cases in which bills for establishing a United States Bank have been vetoed.
  6. Give as complete a chronology as you can of the events connected with the Bank, from President Jackson’s first attack upon it down to its final failure.
  7. The removal of the deposits is sometimes spoken of as a fatal blow to the United States Bank. What do you gather from your reading as to its importance as regards the business position or credit of the Bank?
  8. What was the Specie Circular of 1836, and what serious financial results did it produce?
  9. What led to the adoption of the National Bank system in 1863?
  10. How would it have eased the financial difficulty in 1861, if the Secretary of the Treasury had made more free use of his authority, under the act of August 5, for suspending some of the provisions of the Independent Treasury act?
  11. The earlier legal-tender acts provided for funding the notes, at the pleasure of the holder, in United States bonds. When and why was this privilege of funding withdrawn? What would probably have been the effect if it had been retained until the close of the war?
  12. What were the steps by which the legal tender issues came to be treated as the practically permanent element in our paper currency and to be fixed in amount?

Source: Harvard University Archives. Harvard University, Mid-year examinations, 1852-1943. Box 3. Bound volume Examination Papers, Mid-Year 1894-95.

__________________

1894-95.
ECONOMICS 9.

Course Title, Staffing, and Enrollment

[ECONOMICS] 9. Asst. Professor Cummings. — The Social and Economic Condition of Workingmen in the United States and in other countries. 3 hours.

Total 79: 3 Graduates, 34 Seniors, 31 Juniors, 5 Sophomores, 6 Others.

Source: Harvard University. Report of the President of Harvard College, 1894-1895, p. 62.

 

ECONOMICS 9.
Mid-Year Examination, 1895.

(Arrange your answers, in the order in which the questions stand. So far as possible illustrate your discussions by a comparison of the experience of different countries.)

  1. State accurately the reading you have done in this course to date.
  2. “The interests of the working classes are identical in all lands governed by capitalist methods of production. The extension of the world’s commerce and production for the world’s markets, make the position of the workman in any one country daily more dependent upon that of the workmen in other countries.” Why? Explain how in the history of trade unions this community of interest among workmen, not only of the same trade and the same country but of different trades and different countries, has actually manifested itself. Illustrate.
  3. Precisely what answer to the “lump of labor” theory is to be drawn from that version of the wage-fund doctrine adopted by Mill, by Walker, by yourself?
  4. How far has the theory and the practice of coöperation offered a complete remedy for the evils of the existing industrial organization? and at precisely what points has the theory and the practice broken down? Illustrate carefully.
  5. “The struggle of the working classes against capitalist exploitation must of necessity be a political struggle.” How far does the history of trade unions and of coöperation show a tendency in this direction? Illustrate carefully.
  6. “But above all things, observe that all types of piece wage, whether single or progressive, and whether individual or collective, possess this most marked superiority over Profit-sharing.” … “At the same time, it is right to remark that there are many cases, in which the method of Profit-sharing surpasses in important respects any form of the ordinary wage-system.” Explain carefully the grounds of the alleged inferiority and superiority in each case.
  7. “Before, therefore, the trade union can realize its policy of ‘collective bargaining,’ it must solve the two-fold problem – how to bind its own constituents, and how to obtain the recognition of employers.” By what methods have trade unions endeavored to solve this problem? Illustrate.
  8. Trace the successive stages of the so-called “industrial revolution” during the last hundred and fifty years.

Source: Harvard University Archives. Harvard University, Mid-year examinations, 1852-1943. Box 3. Bound volume Examination Papers, Mid-Year 1894-95.

 

ECONOMICS 9.
Year-end Examination. 1895.

[Arrange your answers in the order in which the questions stand. So far as possible illustrate your discussions by statistical and descriptive matter showing the relative condition of working people in the United States and in other countries.]

I.

State accurately the reading you have done in this course since the mid-year examinations.

II.

Devote three hours to a careful discussion of the merits and defects of the German system of compulsory insurance, under the following general heads:

  1. An accurate account of the origin, scope, organization, administration of the system in Germany, — stating approximately the numbers insured, the cost of insurance to all parties concerned, the benefits provided, the methods of collection, distribution, etc.;
  2. Difficulties, opposition, and criticisms thus far encountered;
  3. Progress of similar movements towards compulsory insurance in other countries;
  4. Facts bearing upon the adequacy of existing provisions for sickness, accident, old age in England and the United States;
  5. A biographical sketch showing at what age and in what respects the State already interferes to prescribe conditions of employment, education, etc., for operatives reared from childhood to old age in the factory system of Massachusetts: showing also the additional interference which would be involved in the adoption of the German system of compulsory insurance;
  6. Conclusion.

Source: Harvard University Archives. Harvard University, Examination Papers, 1873-1915. Box 4. Bound volume: Examination Papers, 1893-95. “Papers Set for Final Examinations in Philosophy, History, Government and Law, Economics Fine Arts, Architecture, and Music in Harvard College, June, 1895,” p. 40.

__________________

1894-95.
ECONOMICS 10.

Course Title, Staffing, and Enrollment

[ECONOMICS] 10. Professor Ashley. — The Elements of Economic History from the Middle Ages to Modern Times. 2 hours.

Total 61: 9 Graduates, 20 Seniors, 21 Juniors, 10 Sophomores, 1 Other.

Source: Harvard University. Report of the President of Harvard College, 1894-1895, p. 62.

 

ECONOMICS 10.
Mid-Year Examination. 1895.

I. To be first attempted by all.

Translate, and comment on, the following passages:

  1. Quomodo vocatur mansio; quis tenuit eam T. R. E.; quis mod tenet; quot hidae; quot carrucae in dominio; quot hominum; quot villani; quot cotarii; quot servi; quot liberi homines; quot sochemanni.
  2. De virgis operantur ii diebus in ebdomada.
  3. Rex. . . destinavit per regnum quos ad id prudentiores.. . . cognoverat, qui circumeuntes et oculata fide fundos singulos perlustrantes, habita aestimatione victualium quae de hiis solvebantur, redegerunt in summam denariorum.
  4. Interiors plerique frumenta non serunt, sed lacte et carne vivunt, pellibusque sunt vestiti.
  5. Ideo rogamus, sacratissime imperator, subvenias. . . . ademptum sit jus etiam procuratoribus, nedum conductori, adversus colonos ampliandi partes agrarias.
  6. Arva per annos mutant et superest ager.
  7. Ego Eddi episcopus terram quae dicitur Lantocal tres cassatos Heglisco abbati libenter largior.
  8. Rex misit in singulos comitatus quod messores et alii operarii non plus caperent quam capere solebant.
  9. Noveritis nos concessisse omnibus tenentibus nostris . . . . quod omnia praedicta terrae et tenementa de cetero sint libera, et liberae conditionis.

II. Write on two only of the following subjects.

  1. The reasons for believing in the survival in Britain of the Roman agrarian organisation.
  2. A comparison, from the economic point of view, of the open-field system with modern methods of farming.
  3. The condition of the tillers of the soil in England in A.D. 1381 as compared with A.D. 1066.

Source: Harvard University Archives. Harvard University, Mid-year examinations, 1852-1943. Box 3. Bound volume Examination Papers, Mid-Year 1894-95.

 

ECONOMICS 10.
Year-end Examination. 1895.

I.
[To be first attempted by all.]

TRANSLATE, and comment on, the following passages:—

  1. In Kateringes sunt x. hidae ad geldum Regis. Et de istis
  2. hidis tenent xl. villani xl. virgas terrae. … Et omnes isti homines operantur iiibus diebus in ebdomada.
  3. Agriculturae non student; majorque pars eorum victus in lacte caseo carne consistit; neque quisquam agri modum certum aut fines habet proprios; sed magistratus ac principes in annos singulos gentibus cognationibusque hominum qui una coierunt, quantum et quo loco visum est agri attribuunt atque anno post alio transire cogunt.
  4. Nul ne deit rien acheter a revendre en la vile meyme, fors yl serra Gildeyn.
  5. Cives Londoniae debent xl marcas pro Gilda Telaria delenda; ita ut de cetero non suscitetur.
  6. Johannes Hore mortuus est, qui tenuit de domino dimidiam acram terrae cujus heriettum unus vitulus precii iiii d. Et Johanna soror dicti Johannis est proximus heres, quae venit et gersummavit dictam terram tenendam sibi et suis in villenagio ad voluntatem per servicia et consuetudines.

II.
[Write on four only of the following subjects.]

  1. “I contend that from 1563 to 1824, a conspiracy, concocted by the law and carried out by parties interested in its success, was entered into, to cheat the English workman of his wages, to tie him to the soil, to deprive him of hope, and to degrade him into irremediable poverty.” Consider this.
  2. Discuss the question whether the statute of 5 Eliz. c. 4, displays any distinct economic policy.
  3. Explain the causes, nature and consequences of the change in commercial routes in the sixteenth century.
  4. What is meant by a national economy, as contrasted with a town economy? Illustrate from European conditions in the 15th and 16th centuries.
  5. What has been the economic gain to England from immigration?
  6. Mention briefly those respects in which the economic development of England has resembled that of Western Europe, and those respects in which it has been peculiar.

Source: Harvard University Archives. Harvard University, Examination Papers, 1873-1915. Box 4. Bound volume: Examination Papers, 1893-95. “Papers Set for Final Examinations in Philosophy, History, Government and Law, Economics Fine Arts, Architecture, and Music in Harvard College, June, 1895,” pp. 40-41.